Set which contains all of it's subsets

In summary: V of all sets is not a set. if we "allow" A∈A, then the class of all sets is a set, and then we can't form the set of all sets, but we can still form the "class" of all sets, and we can form the "subclass" of all sets that are not members of themselves, and these classes are distinct.In summary, the conversation discusses the non-existence of a set that contains all of its subsets. The attempt at a solution involves creating a set P which is defined as {x | x \
  • #1
Berrius
19
0

Homework Statement


Prove a set which contains all of it's subsets doesn't exist.

The Attempt at a Solution


Suppose such a set P exists. P := {x | x [itex]\in[/itex] [itex]\wp[/itex](x)}.
P [itex]\in[/itex] [itex]\wp[/itex](x), so P [itex]\in[/itex] P.
This seems like a paradox to me, so all I have to prove is that a set can't contain itself. But how? I've got a gut feeling it's closely related with Russels paradox, but I can't get it.
 
Physics news on Phys.org
  • #2
Have you proven that the power set of a set is always strictly larger?

There's no need to get involved with fundamental things like Russell's paradox. This is more basic.
 
  • #3
mr. vodka said:
Have you proven that the power set of a set is always strictly larger?

There's no need to get involved with fundamental things like Russell's paradox. This is more basic.
Yes I've proven that there doesn't exist a surjection f: S [itex]\rightarrow[/itex] [itex]\wp[/itex](S), but there exists an injection.
 
  • #4
Say X is the set in your problem, the one that contains all its subsets.

How do X and its power set relate in this case?
 
  • #5
I don't know if the axioms rule out the possibility of a set that is a member of itself. I'm guessing that they don't.

I would avoid constructions of the form {x|x has property P}, because that's what's gets us into trouble in the case of Russell's paradox. The axiom schema of comprehension says that {x∈A|x has property P} is allowed when A is a set, but not that {x|x has property P} is allowed. So I would start in a different way, specifically with the sentence "Suppose that there's a set A such that ##x\subset A\Rightarrow x\in A##.", and then try to derive a contradiction from that.

Hint: Cardinality. (I think that's a good hint, but I haven't made absolutely sure that my idea works).
 
  • #6
mr. vodka said:
Say X is the set in your problem, the one that contains all its subsets.

How do X and its power set relate in this case?
X would be it's power set, and thus there would be a bijection between them. However I've proven there doesn't exist a bijection. In other words, X and it's power set have the same cardinality (if X exists), but I've proven the power set is bigger than X, so there is a contradiction and thus X doesn't exist.

Is this correct?
 
  • #7
Yes :) but as you don't seem to be sure of yourself, let me say it more succinctly: there is no surjective function from X to its power set, which there would be if X equaled its power set.
 
  • #8
Fredrik said:
I don't know if the axioms rule out the possibility of a set that is a member of itself. I'm guessing that they don't.

(snip)

yes, they do. this is a consequnce of the axiom of regularity, and the axiom of pairing:

let A be a set. then {A,A} = {A} is a set, by the axiom of pairing.

by the axiom of regularity, {A} contains an element B such that {A} and B are disjoint. but the only element of {A} is A. therefore, A and {A} are disjoint.

since A is in {A}, A cannot be in A (by the definition of "disjoint").

(if a set could be a member of itself, we could form "the set of all sets", which is a problemmatic construction, and quite similar to, but not quite the same as, Russell's paradox).
 
  • #9
Deveno said:
yes, they do. this is a consequnce of the axiom of regularity, and the axiom of pairing:

let A be a set. then {A,A} = {A} is a set, by the axiom of pairing.

by the axiom of regularity, {A} contains an element B such that {A} and B are disjoint. but the only element of {A} is A. therefore, A and {A} are disjoint.

since A is in {A}, A cannot be in A (by the definition of "disjoint").
Thanks, that proof was very clear. I've been wondering about this.

Deveno said:
if a set could be a member of itself, we could form "the set of all sets"...
I don't see how this implication is true. Because of your proof, it's clear that we can't introduce a set that's a member of itself without first dropping (or weakening) some of the ZF axioms, so suppose that we drop enough of them to make your proof invalid, and then add an axiom that says "there's an A such that A∈A". Does this really imply that there's a set that contains all sets?
 
  • #10
Deveno said:
(if a set could be a member of itself, we could form "the set of all sets", which is a problemmatic construction, and quite similar to, but not quite the same as, Russell's paradox).

ZF with the axiom of regularity is just as consistent as ZF without the axiom. So the axiom of regularity is not the one that prevents problematic constructions. Rather, it is the limited comprehension scheme that prevents a set of all sets, not the regularity axiom.

Furthermore, not all people consider the axiom of regularity as a basic axiom.
 
  • #11
micromass said:
ZF with the axiom of regularity is just as consistent as ZF without the axiom. So the axiom of regularity is not the one that prevents problematic constructions. Rather, it is the limited comprehension scheme that prevents a set of all sets, not the regularity axiom.

Furthermore, not all people consider the axiom of regularity as a basic axiom.

yes, many people see regularity as a "useless axiom" (and some people don't even include it, choosing to use the axiom of infinity instead).

and, regularity does not "solve" Russell's paradox. what regularity says is:

all sets are "well-founded". that is, no set is "infinitely recursively defined in terms of smaller sets". every set "starts somewhere". now there are some good reasons for considering "non-well-founded" set theories (just as the natural numbers are (often) defined impredicatively, but this doesn't really lead to any problems), merging data streams is a good example.

you and Fredrik are correct, however, the "absence of regularity" doesn't imply that there is a "set of all sets". i don't know what i was thinking...probably something along the lines of:

if A∈A is allowable, then for all subsets of the class V of all sets, we could have V∈V...but that doesn't even make sense in NGB, because a∈b is undefined if a is not a set.

*****

however, just because two theories are "equi-consistent" is not, in itself, a reason to favor one or the other. ZFC+CH is equi-consistent with ZFC+(¬CH), which leads one to wonder...should we think of the CH as true, untrue, or both/neither? moreover, sometimes a "minimal axiom system" is less useful than a redundant one: classical logic can be reduced entirely to "nands" or "nors", but doing so obscures the ways we think and speak about things we feel are true (although logical circuits don't seem to mind...).
 

Related to Set which contains all of it's subsets

1. What is a set that contains all of its subsets?

A set that contains all of its subsets is known as a "power set." This means that every possible combination of elements from the original set is included in the power set.

2. How is a power set represented?

A power set is typically represented using curly braces around the original set, with the elements separated by commas. For example, the power set of {1, 2} would be written as {{}, {1}, {2}, {1, 2}}.

3. What is the cardinality of a power set?

The cardinality, or size, of a power set is 2^n, where n is the number of elements in the original set. In other words, if a set has n elements, its power set will have 2^n subsets.

4. How is a power set useful in mathematics?

Power sets are useful in many areas of mathematics, including set theory, combinatorics, and topology. They can also be used in computer science and programming, such as in the implementation of algorithms and data structures.

5. Can a set contain itself as a subset?

No, a set cannot contain itself as a subset. This would violate the axiom of regularity, which states that every set must have an element that is not a subset of itself. Therefore, a set that contains itself as a subset would lead to a contradiction.

Similar threads

  • Calculus and Beyond Homework Help
Replies
1
Views
546
  • Calculus and Beyond Homework Help
Replies
1
Views
1K
  • Calculus and Beyond Homework Help
Replies
4
Views
1K
  • Calculus and Beyond Homework Help
Replies
20
Views
2K
  • Calculus and Beyond Homework Help
Replies
3
Views
1K
  • Calculus and Beyond Homework Help
Replies
4
Views
2K
  • Calculus and Beyond Homework Help
Replies
1
Views
1K
  • Calculus and Beyond Homework Help
Replies
3
Views
2K
  • Calculus and Beyond Homework Help
Replies
11
Views
2K
  • Calculus and Beyond Homework Help
Replies
1
Views
1K
Back
Top